8
$\begingroup$

Fix $n$ and $k$. I want a set $S\subseteq\{1,\ldots,n\}$ with the property that for every $x\in S$,

$$\mathrm{gcd}\bigg(x,\prod_{y\in S\setminus\{x\}}y\bigg)<\frac{x}{k}.$$

How small should a random $S$ be to have this property with high probability? More importantly, what sort of math is this, and where can I learn more? (I only guessed in my tags.)

$\endgroup$
5
  • 1
    $\begingroup$ Is S same as A? $\endgroup$
    – Boris Bukh
    Dec 24, 2013 at 22:58
  • $\begingroup$ If S is the set of primes greater than n/2, you can get your property surely, if the y's chosen are at most n. $\endgroup$ Dec 24, 2013 at 23:21
  • 1
    $\begingroup$ Probabilistic number theory seems like the right kind of math. I also observe that this seems to be an interesting question even for $k=1$. $\endgroup$ Dec 25, 2013 at 3:16
  • 1
    $\begingroup$ The following is a sufficient condition for $S$ to have that property - one that could probably be probabilistically analyzed without too much difficulty. Let $\nu_p(s)$ denote the power of the prime $p$ dividing $s$. Then if $\sum_{s\in S} \nu_p(s) > 2\max_{s\in S} \nu_p(s)$ for every prime $p$, the set $S$ has the required property. $\endgroup$ Dec 25, 2013 at 3:32
  • $\begingroup$ Not sure how rigorous this can be made, but it seems like when $n$ is large, $\nu_p$ of a random member of $\{1,\ldots,n\}$ nearly behaves like a geometric random variable with success probability $1-1/p$. As such, I would think that each member of $S$ will tend to have a large (i.e., uncommon) prime divisor, and the size of $S$ would be dictated by the collision probability of these large primes. $\endgroup$ Dec 25, 2013 at 20:39

2 Answers 2

6
$\begingroup$

Let's just consider the case $k=1$ where the problem asks for sets $S$ such that each element of $S$ does not divide the product of the rest of the elements of $S$. I claim that if $S$ has fewer than $\exp(\frac{1}{10} \sqrt{\log n\log \log n})$ elements then with high probability this happens. On the other hand if $S$ has more than $\exp(10\sqrt{\log n \log \log n})$ elements then with high probability some element of $S$ will divide the product of the remaining elements. With more effort this gap can probably be closed. (Also, as will be seen from the proof the argument also works if $k$ is not too big -- that is in the range $k\le \exp(\frac{1}{10} \sqrt{\log n\log \log n})$ say.)

The reasoning relies on some facts about smooth numbers. Let $\Psi(x,y)$ denote the number of integers up to $x$ all of whose prime factors are below $y$. We are interested in the range when $y$ is about $\exp(c\sqrt{\log x\log \log x})$ for some constant $c$, and here $\Psi(x,y)$ is about size $x\exp(-\sqrt{\log x\log \log x} (\frac{1}{2c}+o(1)))$; moreover the number of squarefree smooth numbers is also of this size.

I need one more observation. If $s$ numbers are chosen randomly from $1$ to $n$, then their product will very likely be divisible by all primes up to $s/\log n$ -- if not there is a prime $p<s/\log n$ not dividing all these $s$ numbers which happens with probability $(1-1/p)^{s} = O(1/n)$, and there are at most $n/\log n$ such primes $p$, for a total probability of at most $1/\log n$. In the other direction, if $s$ numbers (with $s$ reasonably large) are chosen randomly from $1$ to $n$ then their product is very likely not divisible by $p^2$ for every prime $p>s^3$ say. To see this, if we pick a prime $p>s^3$ then the chance that $p^2$ divides the product is $O(s^2/p^2)$ ($p$ could divide two of the $s$ elements; the probability of dividing $3$ elements is even smaller) and summing this over all $p>s^3$ still gives a total probability of $O(1/s)$.

Now we are ready for the proof. Suppose $S$ has fewer than $L_1=\exp(\frac 1{10} \sqrt{\log n\log \log n})$ elements. From our remark on the number of smooth integers, we may see with high probability each element of $S$ is not $L_1^3$ smooth. That is each element of $S$ is likely to be divisible by a prime larger than $|S|^3$. But then as observed above, such a large prime is unlikely to divide two elements of $S$, and therefore with high probability every element of $S$ does not divide the product of the rest.

For the other assertion, suppose now that $S$ has more than $L_2 =\exp(10\sqrt{\log n\log \log n})$ elements. From our observation on smooth numbers, with high probability $S$ contains a squarefree number that is $\exp(\sqrt{\log n\log \log n})$ smooth. But with high probability the product of the remaining elements of $S$ is divisible by all primes up to $(|S|-1)/\log n$ which is a good deal bigger than $\exp(\sqrt{\log n\log \log n})$. In other words, the squarefree smooth number that we are likely to find in $S$ will divide the product of the remaining numbers with high probability.

This completes the proof. Let me add that the shape of the answer $\exp(\sqrt{\log n\log \log n})$ arises in other contexts (e.g. factoring algorithms such as the quadratic sieve) for a similar reason: it is the break even value of $y$ where the probability of being $y$ smooth is roughly $1/y$, and that's roughly what's being used above.

$\endgroup$
3
$\begingroup$

We can see that $x$ must not divide the product of the other elements of $S$ because if it does,$$\mathrm{gcd}\bigg(x,\prod_{y\in S\setminus\{x\}}y\bigg)=x\geq \frac{x}{k}$$ for every $k\geq 1$ .Erdos has proved that every set $S$ with the above property must have $\pi(n)$ elements at most.

"Let $a_n$ be a sequence of positive integers with $1<a_1<\cdots<a_n\leq N$ which has the property:
(A) $a_i\nmid \frac{a_1\cdot a_2\cdots a_n}{a_i}$ for every $i=1,...,n$ .Then $n\leq \pi(N)$ holds"

(As it was asked) here is the
proof: We will see that if a proper choise of maximum number of elements not exceeding $n$ with the mentioned property exists,then we can construct another (equivalent) set of elements containing only prime powers.

Suppose that the maximum number of elements we can choose from $\{2,...,n\}$ with the mentioned property is $r\geq \pi(n)+1$.
It is impossible to have all elements prime powers because by the pigeonhole principle there will be 2 elements $p^a,p^b$ with $a<b$ and $p^a|p^b$ which means that the desired property does not hold for $p^a$.
So,there must be at least one element that can be written as $x=k\cdot m$ with $\gcd (k,m)=1$.

If $k$ does not divide the product of the rest elements and so does $m$, then we can pull out $x$ from the set and place $k$ and $m$ into the set ,having a new set with $r+1$ elements with property (A) holding true.
(of course no other of the elements is equal to $k$ or $m$ because this would again lead to a contradiction)
But this is a contradiction since $r$ is the maximum number of elements as we assumed.

So,without loss of generality we may assume that $k$ does not divide the product of the rest,but $m$ does.
This means that we can replace $x$ with $k$ in the set with property A) holding true.
We repeat the argument again until $k$ "drops" to a prime power.
( which lets us arrive at a contradiction for prime powers as we already mentioned at the beggining)

(By the way this is not Erdos's proof but one i found some years ago.But i am almost sure Erdos proved this theorem)

So, your set must have at most $\pi(n)$ elements.
On the other hand, if your set contains at least $\pi(n)+1$ elements it could contain 2 powers of the same prime,$p,p^m$ and so the gcd you want would be at least $x=p$ (or $x=p^m$).
I think it would be more difficult to determine the asymptotic size of $S$ depending on $k$ but certainly $|S|\leq\pi(n)$ holds for a random set $S$ as you require.

$\endgroup$
6
  • $\begingroup$ Thanks - Can you include a reference? $\endgroup$ Dec 25, 2013 at 12:48
  • $\begingroup$ At the beggining i thought that Erdos's theorem was included in "Topics in the theory of numbers"(Springer) but i could not find it right now.If you wish i could give the proof of the above statement. $\endgroup$ Dec 25, 2013 at 13:08
  • 2
    $\begingroup$ This doesn't really answer the posted question - I suspect a random set must be much smaller to have this property - but it is an interesting fact nonetheless. $\endgroup$ Dec 25, 2013 at 15:54
  • $\begingroup$ @panoramix - Yes, I'd like to see the proof. :) $\endgroup$ Dec 25, 2013 at 18:32
  • 1
    $\begingroup$ @Dustin G. Mixon: this seems to be easy to prove. Suppose that no $s\in S$ divides the product $P(s):=\prod_{t\in S\setminus\{s\}} t$. Then for any $s\in S$ there is a prime $p\in[1,n]$ such that, notation as in Greg Martin's comment, we have $\nu_p(s)>\nu_p(P(s))$. Now, if we had $|S|>\pi(n)$, then there would be two elements of $S$ associated with the same prime, which is a clear nonsense: if, say, $\nu_p(s_1)\le\nu_p(s_2)$, then $\nu_p(s_1)\le\nu_p(P(s_1))$ since $s_2\mid P(s_1)$. $\endgroup$
    – Seva
    Dec 25, 2013 at 19:52

Your Answer

By clicking “Post Your Answer”, you agree to our terms of service and acknowledge you have read our privacy policy.

Not the answer you're looking for? Browse other questions tagged or ask your own question.